Search results

  1. M

    2014 Extension 1 BOS Trial Exam Discussion Thread

    Hahaha, thankyou :) just realised that what I should have done was P(k+1)/P(k) > 1, which is then equivalent to P(k+1) > P(k).
  2. M

    2014 Extension 1 BOS Trial Exam Discussion Thread

    I was wondering whether someone could please give a hint for 13) part b) ii) and iii). Initially I thought to find P(k+1)/P(k) = 1/((n-1)(k+1)(m-k+1)) and find the smallest k for which P(k+1)/P(k) < 0. If k = m+1, then P(k+1)/P(k) = 0 and if k = m + 2, P(k+1)/P(k) = 1/((n-1)(m+2)(-1) < 0 so...
Top